[obm-l] Re: [obm-l] Re: [obm-l] Re: [obm-l] Re: [obm-l] Re: [obm-l] Re: [obm-l] Teoria dos números

2020-03-17 Por tôpico Pedro José
Boa noite!
Aí, como dizia minha falecida vó, são outros quinhentos.
Como nas propostas anteriores n era natural. Vamos seguir nessa linha, se
não for reformule o problema.
Seja f(n)=   n (427 - 90n - 70n^2 + 45n^3 + 18n^4)
f(0)=0 qualquer natural divide, portanto, é indiferente.
f(1)= 330
f(2)= 1230
É fácil verificar que mdc(330,1230)=30 então D<=30, onde D é o máximo
inteiro que divide f(n) para todo n natural.
f(n) = 7n +5n^4 + 8 n^5 mod 10.
f(0)=0 mod10
f(1)= 20 = 0 mod10
f(2)= 350= 0 mod10
f(3)= 2370 = 0 mod10
f(4)= 9500 = 0 mod10
f(5)= 28160 = 0 mod10
f(6)=68730 = 0 mod10
f(7)=146510 = 0 mod10
f(8)=282680 = 0 mod10
f(9)=505260 = 0 mod10
logo 10 | f(n) para qualquer n natural.

f(n) = n -n^3 mod 3
f(0) = 0 mod 3
f(1) = 0 mod 3
f(2)= -6 = 0 mod 3
logo 3| f(n) para todo n natural
então D = 30.

Saudações,
PJMS



Em ter., 17 de mar. de 2020 às 11:57, Israel Meireles Chrisostomo <
israelmchrisost...@gmail.com> escreveu:

> Sim é isso q eu quis dizer
>
> Em ter, 17 de mar de 2020 11:12, Carlos Gustavo Tamm de Araujo Moreira <
> g...@impa.br> escreveu:
>
>> Acho que a pergunta deve ser qual é o maior inteiro positivo que divide
>> essa expressão para todo valor de n ao mesmo tempo.
>>
>> On Tue, Mar 17, 2020 at 6:58 AM Pedro José  wrote:
>>
>>> Bom dia!
>>> Se você considerar a expressão n(427-90n-70n^2+45n^3+18n^4)
>>> D=|n(427-90n-70n^2+45n^3+18n^4)|
>>> Por exemplo, n=1
>>> D=330.
>>> Agora se liberar n para variar D tende a oo.
>>>
>>> Se n for raiz da expressão, também tende a oi, pois qualquer inteiro
>>> divide 0.
>>>
>>>
>>> Em seg, 16 de mar de 2020 22:16, Israel Meireles Chrisostomo <
>>> israelmchrisost...@gmail.com> escreveu:
>>>
 não entendi

 Em seg., 16 de mar. de 2020 às 22:01, Pedro José 
 escreveu:

> Para um dado n é o módulo do valor da expressão.
>
> Em seg, 16 de mar de 2020 21:49, Pedro José 
> escreveu:
>
>> Boa noite!
>> O módulo dessa expressão tende a oo. Não existe máximo.
>> Saudações,
>> PJMS
>>
>> Em seg, 16 de mar de 2020 20:36, Israel Meireles Chrisostomo <
>> israelmchrisost...@gmail.com> escreveu:
>>
>>> Qual é o maior inteiro que divide  n (427 - 90n - 70n^2 + 45n^3 +
>>> 18n^4)?
>>>
>>> --
>>> Israel Meireles Chrisostomo
>>>
>>> --
>>> Esta mensagem foi verificada pelo sistema de antivírus e
>>> acredita-se estar livre de perigo.
>>
>>
> --
> Esta mensagem foi verificada pelo sistema de antivírus e
> acredita-se estar livre de perigo.



 --
 Israel Meireles Chrisostomo

 --
 Esta mensagem foi verificada pelo sistema de antivírus e
 acredita-se estar livre de perigo.
>>>
>>>
>>> --
>>> Esta mensagem foi verificada pelo sistema de antivírus e
>>> acredita-se estar livre de perigo.
>>
>>
>> --
>> Esta mensagem foi verificada pelo sistema de antivírus e
>> acredita-se estar livre de perigo.
>
>
> --
> Esta mensagem foi verificada pelo sistema de antivírus e
> acredita-se estar livre de perigo.

-- 
Esta mensagem foi verificada pelo sistema de antiv�rus e
 acredita-se estar livre de perigo.



[obm-l] Re: [obm-l] Re: [obm-l] Re: [obm-l] Re: [obm-l] Re: [obm-l] Teoria dos números

2020-03-17 Por tôpico Israel Meireles Chrisostomo
Sim é isso q eu quis dizer

Em ter, 17 de mar de 2020 11:12, Carlos Gustavo Tamm de Araujo Moreira <
g...@impa.br> escreveu:

> Acho que a pergunta deve ser qual é o maior inteiro positivo que divide
> essa expressão para todo valor de n ao mesmo tempo.
>
> On Tue, Mar 17, 2020 at 6:58 AM Pedro José  wrote:
>
>> Bom dia!
>> Se você considerar a expressão n(427-90n-70n^2+45n^3+18n^4)
>> D=|n(427-90n-70n^2+45n^3+18n^4)|
>> Por exemplo, n=1
>> D=330.
>> Agora se liberar n para variar D tende a oo.
>>
>> Se n for raiz da expressão, também tende a oi, pois qualquer inteiro
>> divide 0.
>>
>>
>> Em seg, 16 de mar de 2020 22:16, Israel Meireles Chrisostomo <
>> israelmchrisost...@gmail.com> escreveu:
>>
>>> não entendi
>>>
>>> Em seg., 16 de mar. de 2020 às 22:01, Pedro José 
>>> escreveu:
>>>
 Para um dado n é o módulo do valor da expressão.

 Em seg, 16 de mar de 2020 21:49, Pedro José 
 escreveu:

> Boa noite!
> O módulo dessa expressão tende a oo. Não existe máximo.
> Saudações,
> PJMS
>
> Em seg, 16 de mar de 2020 20:36, Israel Meireles Chrisostomo <
> israelmchrisost...@gmail.com> escreveu:
>
>> Qual é o maior inteiro que divide  n (427 - 90n - 70n^2 + 45n^3 +
>> 18n^4)?
>>
>> --
>> Israel Meireles Chrisostomo
>>
>> --
>> Esta mensagem foi verificada pelo sistema de antivírus e
>> acredita-se estar livre de perigo.
>
>
 --
 Esta mensagem foi verificada pelo sistema de antivírus e
 acredita-se estar livre de perigo.
>>>
>>>
>>>
>>> --
>>> Israel Meireles Chrisostomo
>>>
>>> --
>>> Esta mensagem foi verificada pelo sistema de antivírus e
>>> acredita-se estar livre de perigo.
>>
>>
>> --
>> Esta mensagem foi verificada pelo sistema de antivírus e
>> acredita-se estar livre de perigo.
>
>
> --
> Esta mensagem foi verificada pelo sistema de antivírus e
> acredita-se estar livre de perigo.

-- 
Esta mensagem foi verificada pelo sistema de antiv�rus e
 acredita-se estar livre de perigo.



[obm-l] Re: [obm-l] Re: [obm-l] Re: [obm-l] Re: [obm-l] Re: [obm-l] teoria dos números curiosidade

2019-07-04 Por tôpico Israel Meireles Chrisostomo
muito obrigado!!!

Em qui, 4 de jul de 2019 às 09:13, Claudio Buffara <
claudio.buff...@gmail.com> escreveu:

> Considere o seguinte algoritmo:
> Dada a/b (acho q precisa ser entre 0 e 1), tome o menor n1 tal que 1/n1 <=
> a/b.
> Daí, tome o menor n2 tal que 1/n2 <= a/b - 1/n1.
> Daí tome o menor n3 tal que 1/n3 <= a/b - 1/n1 - 1/n2
> Etc...
> Esse processo eventualmente para (quando uma desigualdade <= se torna uma
> igualdade), com:
> a/b = 1/n1 + 1/n2 + 1/n3 + ... + 1/nk, para algum k.
> Resta saber se produz uma fração egípcia (ou seja, se n1 < n2 < n3 < ...)
> e se a fração egípcia resultante é a menor possível.
> Vou pensar melhor é tentar simular alguns casos numa planilha.
>
>
> Enviado do meu iPhone
>
> Em 3 de jul de 2019, à(s) 22:11, Bernardo Freitas Paulo da Costa <
> bernardo...@gmail.com> escreveu:
>
> > On Wed, Jul 3, 2019 at 8:34 PM Claudio Buffara
> >  wrote:
> >> Infinitas.
> >> Basta usar recursivamente a relação  1/n = 1/(n+1) + 1/(n(n+1)), que
> cada vez você obtém uma representação mais longa.
> >> 1/2 = 1/3 + 1/6 = 1/3 + 1/7 + 1/42 = 1/3 + 1/7 + 1/43 + 1/1806 = ...
> >
> > Mais difícil, talvez, seria calcular qual o menor número de termos
> > necessários para representar p/q :)  Será que isso é NP completo?
> >
> > Abraços,
> > --
> > Bernardo Freitas Paulo da Costa
> >
> > --
> > Esta mensagem foi verificada pelo sistema de antivírus e
> > acredita-se estar livre de perigo.
> >
> >
> > =
> > Instruções para entrar na lista, sair da lista e usar a lista em
> > http://www.mat.puc-rio.br/~obmlistas/obm-l.html
> > =
>
> --
> Esta mensagem foi verificada pelo sistema de antivírus e
>  acredita-se estar livre de perigo.
>
>
> =
> Instru�ões para entrar na lista, sair da lista e usar a lista em
> http://www.mat.puc-rio.br/~obmlistas/obm-l.html
> =
>


-- 
Israel Meireles Chrisostomo

-- 
Esta mensagem foi verificada pelo sistema de antiv�rus e
 acredita-se estar livre de perigo.



[obm-l] Re: [obm-l] Re: [obm-l] Re: [obm-l] Re: [obm-l] Re: [obm-l] Re: [obm-l] Teoria dos números

2019-05-04 Por tôpico Pedro José
Bom dia!
Obrigado!
Encontrei uma demonstração, mas não tive bagavem para enrender. Vou ler as
publicações.

Saudações,
PJMS

Em sáb, 4 de mai de 2019 11:57, Anderson Torres <
torres.anderson...@gmail.com escreveu:

> Em seg, 29 de abr de 2019 às 16:38, Pedro José 
> escreveu:
> >
> > Boa tarde!
> > Pelo menos consegui descobrir que se um inteiro z >= não puder ser
> escrito da forma z=4^k (8m+7), com m,k >=0 e m,k inteiros então ele pode
> ser representado por uma soma de três parcelas, todas quadrados perfeitos.
> > Já a demonstração, não consegui compreender.
> >
>
> Essa é a parte chata. Mas tem paper pra caramba!
>
> https://en.wikipedia.org/wiki/Legendre%27s_three-square_theorem
> Legendre's three-square theorem - Wikipedia
> https://www.sciencedirect.com/science/article/pii/0022314X74900249
> A new proof of the three squares theorem - ScienceDirect
> https://brilliant.org/wiki/fermats-sum-of-two-squares-theorem/
> Sum of Squares Theorems | Brilliant Math & Science Wiki
>
> https://mathoverflow.net/questions/223939/proving-legendres-sum-of-3-squares-theorem-via-geometry-of-numbers
> nt.number theory - Proving Legendre's Sum of 3 Squares Theorem via
> Geometry of Numbers - MathOverflow
> https://core.ac.uk/download/pdf/82306476.pdf
> PII: 0022-314X(74)90024-9 - 82306476.pdf
>
> https://www.ams.org/journals/proc/1957-008-02/S0002-9939-1957-0085275-8/S0002-9939-1957-0085275-8.pdf
> S0002-9939-1957-0085275-8.pdf
> http://pollack.uga.edu/finding3squares-6.pdf
> finding3squares-6.pdf
> https://arxiv.org/pdf/0812.0540.pdf
> () - 0812.0540.pdf
>
>
>
> > Saudações,
> > PJMS
> >
> > Em seg, 29 de abr de 2019 às 14:14,  escreveu:
> >>
> >>
> >> Em 29 de abr de 2019 11:37, Pedro José  escreveu:
> >>
> >> Bom dia!
> >>
> >> Gostei desse problema. Fiz um montão de exemplos com números que não
> podem ser escritos como 4^n(8n+7) e todos puderam ser escritos como a soma
> de três quadrados.
> >> Vale para todos? Se sim, alguém poderia indicar uma demonstração?
> >>
> >> Saudações,
> >> PJMS
> >>
> >> Em dom, 7 de abr de 2019 às 16:16, Pedro José 
> escreveu:
> >>
> >> Boa tarde!
> >> Fiquei na dúvida se algoritmo valia para demonstração. Mas salvo engano
> para demonstração de quais números aceitam raízes primitivas usa-se
> algoritmo.
> >> Mas, agora com mais calma, poderia ter usado indução.
> >> 1) Foi provado que não vale para n=0.
> >> 2) Supondo que não vale para n, não valeria para n+1, por absurdo.
> Pois, se valesse, teria que valer para n.
> >> Creio que teria ficado mais elegante.
> >>
> >> Saudações,
> >> PJMS
> >>
> >>
> >> Em dom, 7 de abr de 2019 às 07:41, matematica10complicada <
> profdouglaso.del...@gmail.com> escreveu:
> >>
> >> Obrigado irmão. Está correto sim.
> >> Douglas O.
> >>
> >> Em qui, 4 de abr de 2019 às 19:44, Pedro José 
> escreveu:
> >>
> >> Boa noite!
> >> Estou mal, mesmo. Ao invés de nenhum li qualquer. Tinha simulado dois,
> três, quatro e deram fora, já iria questionar.
> >> Mas vamos lá:
> >> 0^2 = 0 mod8; 1^2 = 1 mod8; 2^2 = 4 mod8 3^2= 1 mod8; 4^2 = 0 mod 8;
> 5^2 = 1 mod 8 6^2 = 4 mod 8 e 7^2 = 1 mod8;
> >> Portanto o quadrado de um número, ou dá 0 ou da 1 ou 4 na equivalência
> mod8.
> >>
> >> Caso n=0 ==> x=8k+7= 7 mod8. Como mod conserva a soma, não há como
> somar 3 parcelas do conjunto, mesmo com repetição, {0,1,4} e obter 7. Então
> n>0
> >>
> >> Para n>0
> >> x = 4^n*(8K+7) ==> x pertence a 2 |N seja x = a^2 + b^2 + c^2 com a, b,
> c pertencentes a |N - {0}. teríamos que ter a,b,c pares ou um deles par e
> dois ímpares.
> >> mas 4 | x ==> x= 0 mod4. Mas se w pertence a 2|N + 1 ==> w^2 = 1 mod4.
> e se y pertence a 2 |N ==> y^2 = 0 mod 4. Como temos dois ímpares e um par
> e como a soma se conserva temos que x = 2 mod4, absurdo. Portanto só sobra
> a, b, c pares Se a,b,c pares podemos escrevê-los como a= 2s; b=2t e c=2u
> com s,t,u naturais.
> >> x = a^2+b^2+c^2= 4(s^2+t^2+u^2) ==> x1 = 4^(n-1) * (8m+7) = s^2+t^2+u^2
> e vale o mesmo raciocínio de que s,t,u são pares e poderão ser escritos
> como s=2f; t=2g; u= 2h, com f, g, h naturais e seguir nesse algoritmo até
> que tenhamos xj=4^0(8m+7)= p^2+q^2+r^2, absurdo. Pois, já vimos que n= 0
> não atende.
> >>
> >> Espero estar correto.
> >>
> >> Saudações.
> >>
> >>
> >>
> >>
> >>
> >> Em qua, 3 de abr de 2019 às 15:36, matematica10complicada <
> profdouglaso.del...@gmail.com> escreveu:
> >>
> >> Mostre que nenhum número da forma (4^n)(8k+7) , com n e k naturais pode
> ser escrito como soma de 3 tres quadrados
> >>
> >> Douglas Oliveira
> >>
> >> --
> >> Esta mensagem foi verificada pelo sistema de antivírus e
> >> acredita-se estar livre de perigo.
> >>
> >>
> >> --
> >> Esta mensagem foi verificada pelo sistema de antivírus e
> >> acredita-se estar livre de perigo.
> >>
> >>
> >> --
> >> Esta mensagem foi verificada pelo sistema de antivírus e
> >> acredita-se estar livre de perigo.
> >>
> >>
> >> --
> >> Esta mensagem foi verificada pelo sistema de antiv�rus e
> >> acredita-se estar livre de perigo.
> >>
> >>
> >>
> >> 

[obm-l] Re: [obm-l] Re: [obm-l] Re: [obm-l] Re: [obm-l] Re: [obm-l] Teoria dos números

2019-05-04 Por tôpico Anderson Torres
Em seg, 29 de abr de 2019 às 16:38, Pedro José  escreveu:
>
> Boa tarde!
> Pelo menos consegui descobrir que se um inteiro z >= não puder ser escrito da 
> forma z=4^k (8m+7), com m,k >=0 e m,k inteiros então ele pode ser 
> representado por uma soma de três parcelas, todas quadrados perfeitos.
> Já a demonstração, não consegui compreender.
>

Essa é a parte chata. Mas tem paper pra caramba!

https://en.wikipedia.org/wiki/Legendre%27s_three-square_theorem
Legendre's three-square theorem - Wikipedia
https://www.sciencedirect.com/science/article/pii/0022314X74900249
A new proof of the three squares theorem - ScienceDirect
https://brilliant.org/wiki/fermats-sum-of-two-squares-theorem/
Sum of Squares Theorems | Brilliant Math & Science Wiki
https://mathoverflow.net/questions/223939/proving-legendres-sum-of-3-squares-theorem-via-geometry-of-numbers
nt.number theory - Proving Legendre's Sum of 3 Squares Theorem via
Geometry of Numbers - MathOverflow
https://core.ac.uk/download/pdf/82306476.pdf
PII: 0022-314X(74)90024-9 - 82306476.pdf
https://www.ams.org/journals/proc/1957-008-02/S0002-9939-1957-0085275-8/S0002-9939-1957-0085275-8.pdf
S0002-9939-1957-0085275-8.pdf
http://pollack.uga.edu/finding3squares-6.pdf
finding3squares-6.pdf
https://arxiv.org/pdf/0812.0540.pdf
() - 0812.0540.pdf



> Saudações,
> PJMS
>
> Em seg, 29 de abr de 2019 às 14:14,  escreveu:
>>
>>
>> Em 29 de abr de 2019 11:37, Pedro José  escreveu:
>>
>> Bom dia!
>>
>> Gostei desse problema. Fiz um montão de exemplos com números que não podem 
>> ser escritos como 4^n(8n+7) e todos puderam ser escritos como a soma de três 
>> quadrados.
>> Vale para todos? Se sim, alguém poderia indicar uma demonstração?
>>
>> Saudações,
>> PJMS
>>
>> Em dom, 7 de abr de 2019 às 16:16, Pedro José  escreveu:
>>
>> Boa tarde!
>> Fiquei na dúvida se algoritmo valia para demonstração. Mas salvo engano para 
>> demonstração de quais números aceitam raízes primitivas usa-se algoritmo.
>> Mas, agora com mais calma, poderia ter usado indução.
>> 1) Foi provado que não vale para n=0.
>> 2) Supondo que não vale para n, não valeria para n+1, por absurdo. Pois, se 
>> valesse, teria que valer para n.
>> Creio que teria ficado mais elegante.
>>
>> Saudações,
>> PJMS
>>
>>
>> Em dom, 7 de abr de 2019 às 07:41, matematica10complicada 
>>  escreveu:
>>
>> Obrigado irmão. Está correto sim.
>> Douglas O.
>>
>> Em qui, 4 de abr de 2019 às 19:44, Pedro José  escreveu:
>>
>> Boa noite!
>> Estou mal, mesmo. Ao invés de nenhum li qualquer. Tinha simulado dois, três, 
>> quatro e deram fora, já iria questionar.
>> Mas vamos lá:
>> 0^2 = 0 mod8; 1^2 = 1 mod8; 2^2 = 4 mod8 3^2= 1 mod8; 4^2 = 0 mod 8; 5^2 = 1 
>> mod 8 6^2 = 4 mod 8 e 7^2 = 1 mod8;
>> Portanto o quadrado de um número, ou dá 0 ou da 1 ou 4 na equivalência  mod8.
>>
>> Caso n=0 ==> x=8k+7= 7 mod8. Como mod conserva a soma, não há como somar 3 
>> parcelas do conjunto, mesmo com repetição, {0,1,4} e obter 7. Então n>0
>>
>> Para n>0
>> x = 4^n*(8K+7) ==> x pertence a 2 |N seja x = a^2 + b^2 + c^2 com a, b, c 
>> pertencentes a |N - {0}. teríamos que ter a,b,c pares ou um deles par e dois 
>> ímpares.
>> mas 4 | x ==> x= 0 mod4. Mas se w pertence a 2|N + 1 ==> w^2 = 1 mod4. e se 
>> y pertence a 2 |N ==> y^2 = 0 mod 4. Como temos dois ímpares e um par e como 
>> a soma se conserva temos que x = 2 mod4, absurdo. Portanto só sobra a, b, c 
>> pares Se a,b,c pares podemos escrevê-los como a= 2s; b=2t e c=2u com s,t,u 
>> naturais.
>> x = a^2+b^2+c^2= 4(s^2+t^2+u^2) ==> x1 = 4^(n-1) * (8m+7) = s^2+t^2+u^2 e 
>> vale o mesmo raciocínio de que s,t,u são pares e poderão ser escritos como 
>> s=2f; t=2g; u= 2h, com f, g, h naturais e seguir nesse algoritmo até que 
>> tenhamos xj=4^0(8m+7)= p^2+q^2+r^2, absurdo. Pois, já vimos que n= 0 não 
>> atende.
>>
>> Espero estar correto.
>>
>> Saudações.
>>
>>
>>
>>
>>
>> Em qua, 3 de abr de 2019 às 15:36, matematica10complicada 
>>  escreveu:
>>
>> Mostre que nenhum número da forma (4^n)(8k+7) , com n e k naturais pode ser 
>> escrito como soma de 3 tres quadrados
>>
>> Douglas Oliveira
>>
>> --
>> Esta mensagem foi verificada pelo sistema de antivírus e
>> acredita-se estar livre de perigo.
>>
>>
>> --
>> Esta mensagem foi verificada pelo sistema de antivírus e
>> acredita-se estar livre de perigo.
>>
>>
>> --
>> Esta mensagem foi verificada pelo sistema de antivírus e
>> acredita-se estar livre de perigo.
>>
>>
>> --
>> Esta mensagem foi verificada pelo sistema de antiv�rus e
>> acredita-se estar livre de perigo.
>>
>>
>>
>> --
>> Esta mensagem foi verificada pelo sistema de antivírus e
>> acredita-se estar livre de perigo.
>
>
> --
> Esta mensagem foi verificada pelo sistema de antivírus e
> acredita-se estar livre de perigo.

-- 
Esta mensagem foi verificada pelo sistema de antiv�rus e
 acredita-se estar livre de perigo.


=
Instru��es para entrar na lista, sair da lista e usar a lista em

[obm-l] Re: [obm-l] Re: [obm-l] Re: [obm-l] Re: [obm-l] Re: [obm-l] Teoria dos números

2018-06-02 Por tôpico Claudio Buffara
Eu acho que o enunciado pede a soma dos elementos simplesmente porque é uma
questão de múltipla escolha.
Já vi isso antes.

E perguntei a proveniência porque me parece muito difícil para ser uma
questão de vestibular. Talvez do ITA ou da OBM (1a fase)...

***

Sobre as soluções, acho interessante que certos "preconceitos" nos levam a
caminhos totalmente distintos.

Por exemplo, ao ver aquela expressão horrorosa, com X, X5 e X5^5, a
primeira coisa que me ocorreu foi resolver numericamente, com uma planilha.
Daí me pareceu natural procurar uma cota inferior e uma cota superior pra
X, já que a planilha é finita...
Foi aí que me ocorreu a ideia de mudar a notação pra N e R, que facilitou
muito a obtenção das cotas.
Tendo obtido a solução numericamente e redigido a mensagem, dei o caso por
encerrado.
Simplesmente não me ocorreu a ideia de manipular algebricamente o N e o R
pra chegar numa expressão mais reveladora, que foi o caminho que o Bernardo
seguiu, e conseguiu resolver o problema sem planilha.

Enfim, acho interessantíssimo este tema de como surgem as idéias na
matemática, mesmo num problema elementar (mas não trivial) como esse.

***

Com relação à observação do Pedro José, me parece que existe uma diferença
entre o "conjunto solução" e "as raízes" de uma equação.

Como, em se tratando de conjuntos, {1} = {1,1,1}, eu diria que, em relação
à equação x^3 - 3x^2 + 3x - 1 = 0, por exemplo,
o conjunto solução é {1} (igual a {1,1,1}) e a soma das raízes é 3.
Ou seja, esta equação tem 3 raízes (iguais) e um conjunto solução unitário.

Sobre retas coincidentes serem paralelas, acho que esta convenção permite
que paralelismo seja uma relação de equivalência.
E ser expulso por causa disso...ou o professor era um idiota ou, num ímpeto
de adolescente, você usou um linguajar forte demais pra defender sua
posição...

Tem uma passagem no Curso de Análise, do Elon, da qual eu nunca me esqueci:
"No caso de conjuntos, escrever A = B significa que A e B são o mesmo
conjunto, ou seja, que A e B possuem os mesmos elementos." Em outras
palavras, A e B são nomes diferentes para a mesma entidade.


[]s,
Claudio.


2018-06-02 18:08 GMT-03:00 Pedro José :

> Boa noite!
> O que também achei legal nesse problema foi o fato do questionamento ser
> quanto a soma dos elementos do conjunto  solução. Embora bem sutil,
> filosoficamente falando é forte. Pois, ela descarta a interpretação de n
> raízes iguais ao invés de uma raiz de multiplicidade n.
> Todas demonstrações de que algo era único, suponha-se que havia dois e se
> provava, por absudo, que eram iguais. Então se são iguais é único.
> Fui expulso de sala de aula, uma feita, no científico, pois, argumentei
> com o professor que não estava correto o conceito de posição relativa de
> duas retas, chamando-as de duas retas paralelas coincidentes. É uma reta só
> e ela é paralela a ela mesma. Nesse exercício a raiz um tem multiplicidade
> 2, e 0 tem multiplicidade 6, mas não interfere na soma. As demais raízes
> têm multiplicidade 1.
> Para os que seguem que a soma das raízes de um polinômios é
> -An-1/An ao invés de o somatório do produto das raízes pela sua
> multiplicidade, daria 1301, o que, considero incorreto.
> Desculpem-me pela lamúria, mas até hoje não me conformo com n entidades
> iguais com n >1. Talvez a expulsão tenha gerado um trauma.
> Saudações,
> PJMS
>
>
> Em Sáb, 2 de jun de 2018 17:29, Claudio Buffara 
> escreveu:
>
>> De onde é este problema?
>> 1a fase de alguma olimpíada?
>>
>> Abs
>>
>> Enviado do meu iPhone
>>
>> Em 2 de jun de 2018, à(s) 16:15, Daniel Quevedo 
>> escreveu:
>>
>> Muito obrigado a todos. De fato com a mudança de variável fica td mais
>> fácil. Não tinha visto isso.Â
>> Obrigado
>>
>> Em sáb, 2 de jun de 2018 às 16:02, Pedro José 
>> escreveu:
>>
>>> Boa tarde.
>>> A limitação para X5 obviamente não inclui 5, foi lambança.Â
>>> Saudações,Â
>>> PJMSÂ
>>>
>>> Em Sáb, 2 de jun de 2018 15:22, Claudio Buffara <
>>> claudio.buff...@gmail.com> escreveu:
>>>
 Para |X| suficientemente grande, X^6 domina a soma dos outros termos.

 Mudando a notação, eu pus N = X e R = X5.

 Então: R^5*N^5 + R*N = N^6 + R^6.

 Caso 1: N >= 0:
 R <= 4, de modo que o lado esquerdo <= N*(1024*N^4 + 4).
 Já o lado direito >= N^6.
 N*(1024*N^4 + 4) < N^6Â
 ==> 1024*N^4 + 4 < N^5Â Â
 ==> 1024 + 4/N^4 < N
 ==> N >= 1025.
 Então, para a equação ser satisfeita, é necessário que N <= 1024.

 Caso 2: N < 0.
 Então o lado esquerdo <= 0 (com igualdade sss R = 0) e o lado direito
 é positivo.
 Logo, a equação não tem soluções com N < 0.

 Com uma planilha, eu achei apenas 5 soluções:
 0, 1, 32, 243, 1024.

 A soma destes três números é 1300.

 []s,
 Claudio.


 2018-06-02 14:10 GMT-03:00 Daniel Quevedo :

> Seja Xn o resto da divisão de X por n. ParavX inteiro a soma de todos
> os elementos do conjunto solução da 

[obm-l] Re: [obm-l] Re: [obm-l] Re: [obm-l] Re: [obm-l] Re: [obm-l] Teoria dos números

2018-04-10 Por tôpico Pedro José
Bom dia!

Dei uma "roubadinha" e achei outra solução, pois veio de trás para a
frente. Veio da observação que nas respostas u=st.

(s-1)(t-1)(u-1) | ust-1 1=2 e só atende quando k(s,t,u) é
inteiro.
Fixando-se duas váriaveis  k é monótona decrescente para a outra; assim
kmax(s) = k(s,s+1,s+2)= (s(s+1)(s+2)-1)/(s-1)s(s+1)>=2, então
s(s+1)(s+2)/s(s-1)(s+1)>2; s < 4.

(s-1)(t-1)(u-1) | ust-1 então: (u-1) | ust-1, então: (u-1) | ust -st; então
(u-1) | st-1

temos que m (u-1) = st -1, (u-1) = (st-1)/m, com m inteiro. Então m < s,
para que u > t.

s=2, só serve m =1==> u=st. donde: (s-1)(t-1)(u-1) | (st)^2-1 ;
(s-1)(t-1)(u-1) | (u-1)(st+1); (s-1)(t-1) | (st+1)

(t-1)| | 2t+1; (t-1) | 3. t-1 <= 3 ==> t=4 (paridade -se uma das incógnitas
for par todas serão e se uma for ímpar todas serão - e t>s)  e u =st=8, que
verificando atende a proposição. (2,4,8)

s=3 há duas opções m=1 ou m=2

com m=1. (s-1)(t-1) | (st+1);  2(t-1) | 3t+1 ; 2(t-1) | t-1 <=4 ==> t=5
(paridade e t>s) c= st = 15 e atende a proposição (3,5,15)

com m=2 u= (st-1)/2 +1 ==> (s-1)(t-1) | ((st-1)/2 +1)st -1; 2(t-1) <=
(9t^2+3t)/2 -1; impossível.

Logo só há as soluções anteriores (2,4,8) e (3,5,15).

Saudações,
PJMS.

Em 26 de março de 2018 10:49, Pedro José  escreveu:

> Bom dia!
> Agora estou contente. Posso alardear que pelo menos matei um problema da
> IMO.
>
> (s-1)(t-1)(u-1) | ust-1 1 k(s,t,u) = (stu-1)/(s-1)(t-1)(u-1)
>
> Como k(s,t,u) > 1 temos que k(s,t,u) >=2 e só atende quando k(s,t,u) é
> inteiro.
> Fixando-se duas váriaveis  k é monótona decrescente para a outra; assim
> kmax(s) = k(s,s+1,s+2)= (s(s+1)(s+2)-1)/(s-1)s(s+1)>=2, então
> s(s+1)(s+2)/s(s-1)(s+1)>2; s < 4.
>
> fazendo um estudo de paridade: se uma das variáveis for par as outras duas
> também serão e k será ímpar. Se uma das variáveis for ímpar, todas serão
> ímpares e k poderá ser tanto ímpar quanto par.
>
> u   s   v  k
> P  P   P I
> III  -
>
> s=2. k>=3 Para kmax (2,t) = k(2,t,t+1) = (2t(t+1)-1)/t(t-1)>=3 então:
> 2t(t+1)/t(t-1) >3 : t < 5, pela paridade t=4 e kmax(2,4) = 47/15, só serve
> k = 3.
>
> s=2, t=4 e k=3 temos v=8. (2,4,8)
>
> s=3 k>=2 Para kmax (3,t) = k(3,t,t+1) = (3t(t+1)-1)/2t(t-1)>=2 então:
> 3t(t+1)/2t(t-1) >2 : t < 7, pela paridade t=5 e kmax(3,5) = 13/6, só serve
> k = 2.
>
> s=3, t= 5 e k=2 temos v= 15. (3,5,15)
>
> Só atendem: (2,4,8) e (3,5,15)
>
> Achei curioso que em ambas soluções, u=st.
>
> Saudações,
> PJMS
>
>
>
> Em 26 de março de 2018 09:44, Matheus Secco 
> escreveu:
>
>> De fato, trata-se do problema 1 da IMO 1992.
>>
>> Abs,
>>
>> Matheus Secco
>>
>> Em Seg, 26 de mar de 2018 09:24, Claudio Buffara <
>> claudio.buff...@gmail.com> escreveu:
>>
>>> Muito fácil pra ser de IMO...
>>>
>>> 2018-03-26 6:58 GMT-03:00 Anderson Torres 
>>> :
>>>
 Este não é o problema de alguma IMO não? Eu lembro de ter resolvido,
 quase igual à solução oficial: substituir s,t,u por a+1,b+1,c+1 e
 calcular os possiveis valores de
 1/a+1/b+1/c + 1/ab+1/ac+1/bc usando desigualdades - para daí limitar
 os valores de a,b,c.

 Em 23 de março de 2018 17:01, Claudio Buffara
  escreveu:
 > Enfim, nesse meio tempo acho que resolvi o problema...
 >
 > Devemos achar inteiros s, t, u, com 1 < s < t < u e tais que:
 > (stu -1)/((s-1)(t-1)(u-1)) = k  (k inteiro positivo)
 >
 > Após diversas aplicações do truque (método?) de somar e subtrair a
 mesma
 > coisa, chegamos a:
 > stu - 1 =  (s-1)(t-1)(u-1) + (s-1)(t-1) + (s-1)(u-1) + (t-1)(u-1) +
 (s-1) +
 > (t-1) + (u-1)
 >
 > Dividindo isso por (s-1)(t-1)(u-1), obtemos:
 > 1 + 1/(u-1) + 1/(t-1) + 1/(s-1) + 1/((t-1)(u-1)) + 1/((s-1)(u-1)) +
 > 1/((s-1)(t-1)) = k ==>
 >
 > 1/(u-1) + 1/(t-1) + 1/(s-1) + 1/((t-1)(u-1)) + 1/((s-1)(u-1)) +
 > 1/((s-1)(t-1)) = k-1
 >
 > Agora a ideia é achar cotas para s e para k.
 >
 > 1 < s < t < u ==> s >= 2, t >= 3 e u >= 4 ==> o lado esquerdo é menor
 ou
 > igual que:
 > 1/3 + 1/2 + 1 + 1/6 + 1/3 + 1/2 = 2+5/6
 >
 > Ou seja, como o lado esquerdo é inteiro (e positivo), só poderá ser
 igual a
 > 1 ou a 2 ==> k = 2 ou k = 3.
 >
 > Se s >= 4, então t >= 5 e u >= 6, e o lado esquerdo será, no máximo,
 igual
 > a:
 > 1/5 + 1/4 + 1/3 + 1/20 + 1/15 + 1/12 < 1.
 >
 > Logo, devemos ter s = 2 ou s = 3.
 >
 > s = 2 ==>
 > 1/(u-1) + 1/(t-1) + 1 + 1/((t-1)(u-1)) + 1/(u-1) + 1/(t-1) = k-1 ==>
 > 2/(t-1) + 2/(u-1) + 1/((t-1)(u-1)) = k-2 ==>
 > Como k-2 deve ser inteiro positivo, k só pode ser 3 e, portanto:
 > 2/(t-1) + 2/(u-1) + 1/((t-1)(u-1)) = 1 ==>
 > (2 + 1/(t-1))/(u-1) = 1 - 2/(t-1) ==>
 > u = 1 + (2t - 1)/(t - 3) = 3 + 5/(t-3) ==>
 > t = 4 e u = 8   ou   t = 8 e u = 4 (não serve pois t deve ser menor
 do que
 > u)
 >
 > s = 3 ==>
 > 1/(u-1) + 

[obm-l] Re: [obm-l] Re: [obm-l] Re: [obm-l] Re: [obm-l] Re: [obm-l] Teoria dos números

2018-03-26 Por tôpico Pedro José
Bom dia!
Agora estou contente. Posso alardear que pelo menos matei um problema da
IMO.

(s-1)(t-1)(u-1) | ust-1 1=2 e só atende quando k(s,t,u) é
inteiro.
Fixando-se duas váriaveis  k é monótona decrescente para a outra; assim
kmax(s) = k(s,s+1,s+2)= (s(s+1)(s+2)-1)/(s-1)s(s+1)>=2, então
s(s+1)(s+2)/s(s-1)(s+1)>2; s < 4.

fazendo um estudo de paridade: se uma das variáveis for par as outras duas
também serão e k será ímpar. Se uma das variáveis for ímpar, todas serão
ímpares e k poderá ser tanto ímpar quanto par.

u   s   v  k
P  P   P I
III  -

s=2. k>=3 Para kmax (2,t) = k(2,t,t+1) = (2t(t+1)-1)/t(t-1)>=3 então:
2t(t+1)/t(t-1) >3 : t < 5, pela paridade t=4 e kmax(2,4) = 47/15, só serve
k = 3.

s=2, t=4 e k=3 temos v=8. (2,4,8)

s=3 k>=2 Para kmax (3,t) = k(3,t,t+1) = (3t(t+1)-1)/2t(t-1)>=2 então:
3t(t+1)/2t(t-1) >2 : t < 7, pela paridade t=5 e kmax(3,5) = 13/6, só serve
k = 2.

s=3, t= 5 e k=2 temos v= 15. (3,5,15)

Só atendem: (2,4,8) e (3,5,15)

Achei curioso que em ambas soluções, u=st.

Saudações,
PJMS



Em 26 de março de 2018 09:44, Matheus Secco 
escreveu:

> De fato, trata-se do problema 1 da IMO 1992.
>
> Abs,
>
> Matheus Secco
>
> Em Seg, 26 de mar de 2018 09:24, Claudio Buffara <
> claudio.buff...@gmail.com> escreveu:
>
>> Muito fácil pra ser de IMO...
>>
>> 2018-03-26 6:58 GMT-03:00 Anderson Torres :
>>
>>> Este não é o problema de alguma IMO não? Eu lembro de ter resolvido,
>>> quase igual à solução oficial: substituir s,t,u por a+1,b+1,c+1 e
>>> calcular os possiveis valores de
>>> 1/a+1/b+1/c + 1/ab+1/ac+1/bc usando desigualdades - para daí limitar
>>> os valores de a,b,c.
>>>
>>> Em 23 de março de 2018 17:01, Claudio Buffara
>>>  escreveu:
>>> > Enfim, nesse meio tempo acho que resolvi o problema...
>>> >
>>> > Devemos achar inteiros s, t, u, com 1 < s < t < u e tais que:
>>> > (stu -1)/((s-1)(t-1)(u-1)) = k  (k inteiro positivo)
>>> >
>>> > Após diversas aplicações do truque (método?) de somar e subtrair a
>>> mesma
>>> > coisa, chegamos a:
>>> > stu - 1 =  (s-1)(t-1)(u-1) + (s-1)(t-1) + (s-1)(u-1) + (t-1)(u-1) +
>>> (s-1) +
>>> > (t-1) + (u-1)
>>> >
>>> > Dividindo isso por (s-1)(t-1)(u-1), obtemos:
>>> > 1 + 1/(u-1) + 1/(t-1) + 1/(s-1) + 1/((t-1)(u-1)) + 1/((s-1)(u-1)) +
>>> > 1/((s-1)(t-1)) = k ==>
>>> >
>>> > 1/(u-1) + 1/(t-1) + 1/(s-1) + 1/((t-1)(u-1)) + 1/((s-1)(u-1)) +
>>> > 1/((s-1)(t-1)) = k-1
>>> >
>>> > Agora a ideia é achar cotas para s e para k.
>>> >
>>> > 1 < s < t < u ==> s >= 2, t >= 3 e u >= 4 ==> o lado esquerdo é menor
>>> ou
>>> > igual que:
>>> > 1/3 + 1/2 + 1 + 1/6 + 1/3 + 1/2 = 2+5/6
>>> >
>>> > Ou seja, como o lado esquerdo é inteiro (e positivo), só poderá ser
>>> igual a
>>> > 1 ou a 2 ==> k = 2 ou k = 3.
>>> >
>>> > Se s >= 4, então t >= 5 e u >= 6, e o lado esquerdo será, no máximo,
>>> igual
>>> > a:
>>> > 1/5 + 1/4 + 1/3 + 1/20 + 1/15 + 1/12 < 1.
>>> >
>>> > Logo, devemos ter s = 2 ou s = 3.
>>> >
>>> > s = 2 ==>
>>> > 1/(u-1) + 1/(t-1) + 1 + 1/((t-1)(u-1)) + 1/(u-1) + 1/(t-1) = k-1 ==>
>>> > 2/(t-1) + 2/(u-1) + 1/((t-1)(u-1)) = k-2 ==>
>>> > Como k-2 deve ser inteiro positivo, k só pode ser 3 e, portanto:
>>> > 2/(t-1) + 2/(u-1) + 1/((t-1)(u-1)) = 1 ==>
>>> > (2 + 1/(t-1))/(u-1) = 1 - 2/(t-1) ==>
>>> > u = 1 + (2t - 1)/(t - 3) = 3 + 5/(t-3) ==>
>>> > t = 4 e u = 8   ou   t = 8 e u = 4 (não serve pois t deve ser menor do
>>> que
>>> > u)
>>> >
>>> > s = 3 ==>
>>> > 1/(u-1) + 1/(t-1) + 1/2 + 1/((t-1)(u-1)) + 1/(2(u-1)) + 1/(2(t-1)) =
>>> k-1 ==>
>>> > (3/2)/(u-1) + (3/2)/(t-1) + 1/((t-1)(u-1)) = k - 3/2 ==>
>>> > 3/(u-1) + 3/(t-1) + 2/((t-1)(u-1)) = 2k - 3 ==>
>>> > (3 + 2/(t-1))/(u-1) = 2k - 3t/(t-1) ==>
>>> > (3t - 1)/(u-1) = 2k(t-1) - 3t ==>
>>> > u = 1 + (3t - 1)/((2k-3)t - 2k)
>>> >
>>> > k = 2 ==> u = 1 + (3t-1)/(t-4) = 4 + 11/(t-4) ==> t = 5 e u = 15
>>> >
>>> > k = 3 ==> u = 1 + (3t-1)/(3t-6) = 2 + 5/(3t-6) ==> XXX
>>> >
>>> > As únicas soluções são:
>>> > (2,4,8) e (3,5,15)
>>> >
>>> > []s,
>>> > Claudio.
>>> >
>>> > 2018-03-23 15:38 GMT-03:00 Pedro José :
>>> >>
>>> >> Boa tarde!
>>> >>
>>> >> Aproveitando que deu o que falar o problema postado pelo Douglas, tem
>>> um
>>> >> que achei mais interessante.
>>> >>
>>> >> (s-1)(t-1).(u-1) | stu -1, com s, t, u inteiros  e 1 > >>
>>> >> Saudações,
>>> >> Pedro
>>> >>
>>> >> --
>>> >> Esta mensagem foi verificada pelo sistema de antivírus e
>>> >> acredita-se estar livre de perigo.
>>> >
>>> >
>>> >
>>> > --
>>> > Esta mensagem foi verificada pelo sistema de antivírus e
>>> > acredita-se estar livre de perigo.
>>>
>>> --
>>> Esta mensagem foi verificada pelo sistema de antivírus e
>>>  acredita-se estar livre de perigo.
>>>
>>>
>>> 
>>> =
>>> Instru�ões para entrar na lista, sair da lista e usar a lista em
>>> http://www.mat.puc-rio.br/~obmlistas/obm-l.html
>>> 

[obm-l] Re: [obm-l] Re: [obm-l] Re: [obm-l] Re: [obm-l] Re: [obm-l] Teoria dos números

2018-03-26 Por tôpico Anderson Torres
Em 23 de março de 2018 10:35, Claudio Buffara
 escreveu:
> Na verdade os meus questionamentos surgiram por causa do meu interesse em
> ensino de matemática.
>
> Por exemplo, produtos notáveis e fatorações são notoriamente mal ensinados,
> pelo menos nos livros didáticos de 8o e 9o ano que eu examinei.

Eu acho que uma motivação mais geométrica pode ser bastante útil para
muitos produtos notáveis.

Por exemplo, a diferença de quadrados é bem facilmente explicada de
forma geométrica:
um quadrado com um quadradinho a menos no canto pode ser quebrado em
dois trapézios que formam
um retângulo.
O quadrado da soma é mais fácil ainda.

Por outro lado, eu não penso que minha solução foi a mais mágica de
todas, apenas era desconhecida.
Sempre que noto uma expressão simétrica, eu penso em como escrevê-la
em função ou dos
polinômios simétricos elementares ou da soma de potências (x^k+y^k+z^k).

> Nenhum menciona que:
> a) as generalizações de (x+y)^2 = x^2 + 2xy + y^2  e  x^2 - y^2 = (x-y)(x+y)
> para expoentes maiores levam ao teorema do binômio (erroneamente chamado de
> binômio de Newton - nota histórica: Newton generalizou o teorema para
> expoentes racionais) e à fórmula da soma dos termos de uma PG;
> b) (x+y)^2 = x^2 + 2xy + y^2 é a base para a ideia de se completar
> quadrados, a qual, por sua vez, não só resulta na fórmula para as raízes de
> uma equação quadrática, mas também na elucidação das propriedades da função
> quadrática;
> c) o uso inteligente da expansão de (x+y)^3 leva à formula das raízes de uma
> equação cúbica.

Essa eu não conhecia. Ainda penso que as formas mais naturais de lidar
com a cúbica são o Método Gugu-Euler
(tentar uma solução da forma x=raizcúbica(y1)+raizcúbica(y2)) ou usar
a fatoração de x^3+y^3+z^3-3xyz.

Mas o cubo da soma, per se? Isso me parece mágico demais.

(Acho que até imagino o que seja: identificar o termo constante com a
soma de cubos e o termo "linear" com o triplo do produto...)

>
> ***
>
> Há tempos, o Hermann, participante desta lista, postou uma dúvida sobre
> produtos notáveis e pediu dicas de livros com exercícios sobre produtos
> notáveis e fatoração.
> Eu tenho duas sugestões, ambas em inglês:
> - Algebra, de I.M.Gelfand e A.Shen - Birkhäuser (este faz as generalizações
> que eu mencionei acima)
> - A Problem Book in Algebra, de V.A. Krechmar - Mir Publishers (pros
> entusiastas)
> Ambos estão disponíveis na Amazon.
>
> ***
>
> Anos atrás eu gostava de soluções "mágicas", obtidas por meio de alguma
> sacada brilhante que eu jamais conseguiria ter.
> Após me deparar com várias destas soluções, me ocorreu que elas talvez
> tivessem um efeito perverso na motivação dos estudantes de matemática, pois
> passavam a impressão de que é preciso ser um gênio para dominar a matéria.
> Daí o meu interesse em saber como vocês obtiveram certas fatorações.
> Entendo que trabalho braçal, experiência, alguma lógica e um pouco de
> otimismo são, para a maioria de nós, as únicas formas de progredir na
> resolução de um problema como o que deu origem a este thread.
>
> Dito isso (e posso estar enganado) nem o Pedro José e nem mais ninguém
> explicou de onde veio a conjectura (correta) de que:
> z = -(x+y)/2 é solução de (x + y)(y + z)(z + x)/2 + (x + y + z)3 =  – xyz
>
> []s,
> Claudio.
>
>
>
> 2018-03-23 6:20 GMT-03:00 Anderson Torres :
>>
>> Em 21 de março de 2018 09:47, Claudio Buffara
>>  escreveu:
>> > Como você passou de:
>> > 4abc + (a+b+c)^3 + (-a+b+c)(a-b+c)(a+b-c) = 1
>> >
>> > Para:
>> > 4(a+b+c)(ab+ac+bc) - 4abc = 1
>>
>> It's kind of magic. Eu simplesmente abri tudo com vontade e notei
>> certas repetições
>> que sempre aparecem em certas fatorações; ou melhor dizendo, estava
>> pensando em
>> escrever tudo em termos dos famigerados polinômios simétricos e cheguei
>> nisso.
>>
>> Sempre que vejo algo como (a^2b+ab^2), já escrevo ab(a+b) e tento
>> procurar um abc
>> para isso resultar em ab(a+b+c).
>>
>> Mas não avancei daí. Penso que dá para fatorar ainda mais...
>>
>> >
>> > ???
>> >
>> > []s,
>> > Claudio.
>> >
>> >
>> > 2018-03-20 23:14 GMT-03:00 Anderson Torres
>> > :
>> >>
>> >> Em 13 de março de 2018 20:19, Douglas Oliveira de Lima
>> >>  escreveu:
>> >> > Essa achei legal e estou postando.
>> >> >
>> >> > Resolva nos inteiros a seguinte equação:  (x + y)(y + z)(z + x)/2 +
>> >> > (x +
>> >> > y +
>> >> > z)3 = 1 – xyz .
>> >> >
>> >>
>> >> Substituição mágica: x=-a+b+c, y=a-b+c, z=a+b-c. Com isso, x+y=2c,
>> >> x+y+z=a+b+c e
>> >>
>> >> 4abc + (a+b+c)^3 + (-a+b+c)(a-b+c)(a+b-c) = 1
>> >>
>> >> Usando polinômios simétricos,
>> >>
>> >> 4(a+b+c)(ab+ac+bc) - 4abc = 1
>> >>
>> >> Agora estou confuso...
>> >>
>> >> > Abraço do
>> >> > Douglas Oliveira
>> >> >
>> >> > --
>> >> > Esta mensagem foi verificada pelo sistema de antivírus e
>> >> > acredita-se estar livre de perigo.
>> >>
>> >> --
>> >> Esta mensagem foi 

[obm-l] Re: [obm-l] Re: [obm-l] Re: [obm-l] Re: [obm-l] Re: [obm-l] Teoria dos números

2018-03-23 Por tôpico Claudio Buffara
Você não havia explicado que* "fui fazer um experimento tirando o "1" da
equação. Usei um par (x,y) com a mesma paridade e achei um z inteiro.
Novamente usei outro par e deu outro z inteiro. Olhando para os
experimentos. Vi que nos  dois casos z = -(x+y)/2. Ai tornou-se uma
conjectura."*

Ou seja, experimentos numéricos levaram à conjectura. Beleza!

De toda forma, fazer experimentos tirando o "1" da equação foi uma bela
sacada. Você primeiro examinou a versão "homogênea". Boa ideia.

[]s,
Claudio.




2018-03-23 14:30 GMT-03:00 Pedro José :

> Boa tarde!
>
> Cláudio,
> desculpe-me discordar, mas eu disse  de onde veio. Só não veio de nenhuma
> técnica.
> Estava vendo que a parcela do problema: (x+y) (x+z) (y+z)/2 sempre seria
> inteira pois dois desses valores teriam paridade iguais.
> Aí fui fazer um experimento tirando o "1" da equação. Usei um par (x,y)
> com a mesma paridade e achei um z inteiro. Novamente usei outro par e deu
> outro z inteiro. Olhando para os experimentos. Vi que nos  dois casos z =
> -(x+y)/2. Ai tornou-se uma conjectura. Pura sorte.
> Mas, no braço, desenvolvi para para [x,y, -(x+y)/2] e atendeu para essa
> família, aqui deixou de ser conjectura. Foi provado, na grosseria, por
> substituição mas foi.
>
> Saudações,
> PJMS
>
> Em 23 de março de 2018 11:07, Pedro José  escreveu:
>
>> Bom dia!
>> Anderson,
>> o Gugu já avançou, em uma nota acima. E é passível.
>> Revendo a solução do Ralph, fica claro que essa transformação seria de
>> valia.
>> Pois essa transformação leva a :
>> a = (y+z)/2
>> b=  (x+z)/2
>> c= (x+y)/2
>>
>> Então na ordem que o Ralph apresentou: 1/2*(2x+y+z)(x+2y+z)(x+y+2z)=1
>>
>> (b+c) dá a metade do primeiro fator (excetuando-se o fator 1/2) , (a+c) a
>> metade do segundo e (a+b) a metade do terceiro.
>>
>> Saudações,
>>
>>
>>
>> Em 23 de março de 2018 06:20, Anderson Torres <
>> torres.anderson...@gmail.com> escreveu:
>>
>>> Em 21 de março de 2018 09:47, Claudio Buffara
>>>  escreveu:
>>> > Como você passou de:
>>> > 4abc + (a+b+c)^3 + (-a+b+c)(a-b+c)(a+b-c) = 1
>>> >
>>> > Para:
>>> > 4(a+b+c)(ab+ac+bc) - 4abc = 1
>>>
>>> It's kind of magic. Eu simplesmente abri tudo com vontade e notei
>>> certas repetições
>>> que sempre aparecem em certas fatorações; ou melhor dizendo, estava
>>> pensando em
>>> escrever tudo em termos dos famigerados polinômios simétricos e cheguei
>>> nisso.
>>>
>>> Sempre que vejo algo como (a^2b+ab^2), já escrevo ab(a+b) e tento
>>> procurar um abc
>>> para isso resultar em ab(a+b+c).
>>>
>>> Mas não avancei daí. Penso que dá para fatorar ainda mais...
>>>
>>> >
>>> > ???
>>> >
>>> > []s,
>>> > Claudio.
>>> >
>>> >
>>> > 2018-03-20 23:14 GMT-03:00 Anderson Torres <
>>> torres.anderson...@gmail.com>:
>>> >>
>>> >> Em 13 de março de 2018 20:19, Douglas Oliveira de Lima
>>> >>  escreveu:
>>> >> > Essa achei legal e estou postando.
>>> >> >
>>> >> > Resolva nos inteiros a seguinte equação:  (x + y)(y + z)(z + x)/2 +
>>> (x +
>>> >> > y +
>>> >> > z)3 = 1 – xyz .
>>> >> >
>>> >>
>>> >> Substituição mágica: x=-a+b+c, y=a-b+c, z=a+b-c. Com isso, x+y=2c,
>>> >> x+y+z=a+b+c e
>>> >>
>>> >> 4abc + (a+b+c)^3 + (-a+b+c)(a-b+c)(a+b-c) = 1
>>> >>
>>> >> Usando polinômios simétricos,
>>> >>
>>> >> 4(a+b+c)(ab+ac+bc) - 4abc = 1
>>> >>
>>> >> Agora estou confuso...
>>> >>
>>> >> > Abraço do
>>> >> > Douglas Oliveira
>>> >> >
>>> >> > --
>>> >> > Esta mensagem foi verificada pelo sistema de antivírus e
>>> >> > acredita-se estar livre de perigo.
>>> >>
>>> >> --
>>> >> Esta mensagem foi verificada pelo sistema de antivírus e
>>> >>  acredita-se estar livre de perigo.
>>> >>
>>> >>
>>> >> 
>>> =
>>> >> Instru�ões para entrar na lista, sair da lista e usar a lista em
>>> >> http://www.mat.puc-rio.br/~obmlistas/obm-l.html
>>> >> 
>>> =
>>> >
>>> >
>>> >
>>> > --
>>> > Esta mensagem foi verificada pelo sistema de antivírus e
>>> > acredita-se estar livre de perigo.
>>>
>>> --
>>> Esta mensagem foi verificada pelo sistema de antivírus e
>>>  acredita-se estar livre de perigo.
>>>
>>>
>>> 
>>> =
>>> Instru�ões para entrar na lista, sair da lista e usar a lista em
>>> http://www.mat.puc-rio.br/~obmlistas/obm-l.html
>>> 
>>> =
>>>
>>
>>
>
> --
> Esta mensagem foi verificada pelo sistema de antivírus e
> acredita-se estar livre de perigo.
>

-- 
Esta mensagem foi verificada pelo sistema de antiv�rus e
 acredita-se estar livre de perigo.



[obm-l] Re: [obm-l] Re: [obm-l] Re: [obm-l] Re: [obm-l] Re: [obm-l] Re: [obm-l] Teoria dos números

2018-03-20 Por tôpico Claudio Buffara
Seu orgulho talvez seja justificado!

Como você descobriu que qualquer terno ordenado da forma ( x , y , -(x+y)/2
) é solução da equação "sem o 1"?
Isso não me parece nem um pouco óbvio.

Eu sei que, dados três inteiros, pelo menos dois devem ter a mesma
paridade, e que, como a equação é simétrica em x,y,z, podemos supor spdg
que x e y têm a mesma paridade.
Mas daí a termos z = -(x+y)/2 é um salto bastante longo.

Além disso, supor uma solução com  z = -(x+y)/2 + h  para a equação
original (com o 1) também me parece uma sacada brilhante, ainda que leve a
um "salseiro".

[]s,
Claudio.




2018-03-20 16:33 GMT-03:00 Pedro José :

> Boa tarde!
>
> Ralph,
> parabéns pela sua resolução.
> Já, eu, caminhei por caminhos bem mais tortuosos.
> Embora extremamente deselegante é uma solução.
>
> Se xo,yo,zo é uma solução, temos que pelo menos duas incógnitas têm a
> mesma paridade.
> Como o problema é simétrico, sem perda de generalidade, vamos supor que xo
> e yo tenham a mesma paridade.
> então podemos escrever zo = -(xo+yo)/2 + h, com h inteiro.
> Sabendo-se que : [xo,yo, -(xo+yo)/2] é solução da equação sem o "1".
>
> e substituindo na equação original:
>
> h^3 + 2(xo+yo)h^2+ (3/4 (x+y)^2 + xy) h - 1=0.
>
> Como os coeficientes são inteiros as únicas possibilidades de h inteiro
> são 1 e -1.
>
> h=1.
>
> Seja a= xo +yo
>
> 3a^2 + 8a + 4xoyo=0
>
> xoyo >0, temos que 8|a| > 3a^2 ==> |a| < 8/3 ==>|a|=2, portanto xo= -1 e
> yo=-1(não podem ser positivos). Temos z=-(xo+yo)/2+h=0. (-1,-1,2) e suas
> permutações são soluções.
>
> xo.yo = 0 temos a=0 ou a= -8/3 (não atende) ==> xo=yo=0, z= 1. (0,0,1) e
> suas permutações são soluções.
>
> xo.y0 <0
>
> 3a^2 + 8a + 4xoyo=0
>
> para ter solução a inteiro:Δ = (8 + 6i)^2  ==> 64 - 48 xoyo =  64 + 96 i
> + 36 i^2 ==> xoyo = - (2i + 3î^2/4), com i par.
>
> a= i e xoyo = -(2i+ 3i^2/4) então xo e yo são soluções da equação t^2 -it
> - (2i +3i^2/4) = 0. i =2k; t^2 -2kt-(4k+3k^2)=0
>
> Δ = 4k^2 +16k + 12k^2 = 16k(k+1), que nunca será um quadrado perfeito com
> k<>0. Então não há soluções inteiras. (k=0, recaí em xoyo=0)
>
> h=-1
>
> Seja a= xo+yo
>
> -3a^2 +8a -( 4xoyo - 8) = 0
>
> 4xoyo> 8 ==> 8a >3a^2; a <=2; absurdo não atende 4xoyo>8.
>
> 4xoyo-8=0
>
> temos que a=o, não há inteiros que xoyo=2 e xo+yo=0.
>
> 4xoyo - 8 < 0
>
> Δ = (8 + 6i)^2 ; 64 - 48xoyo + 32 = 64 + 96 i + 36 i^2;  xoyo = -2/3 + 2i
> + 3/4i^2, xoyo não pertence aos inteiros não há solução.
>
>
> Ficam apenas: (0,0,1) ; (0,1,0); (1,0,0) ; (-1,-1,2); (-1,2;-1); (2,-1,-1)
>
> Ralph,
>
> Fiz esse salseiro todo, ao invés de fatorar. E olha, que ontem estava
> orgulhoso de ter achado a solução.
>
>
> Saudações,
> PJMS
>
>
>
> Em 20 de março de 2018 12:10, Pedro José  escreveu:
>
>> É acabou me ajudando. Resolvi de uma outra forma, mais complicada, usando
>> a fórmula. Quando tiver um tempo eu posto.
>>
>>
>> Em 19 de mar de 2018 21:02, "Ralph Teixeira" 
>> escreveu:
>>
>>> Opa, opa, opa! Pedro, voce achou uma formula assim generica, z=-(x+y)/2,
>>> que resolve esta equacao? Beleza, excelente ideia, temos um caminho!
>>>
>>> Porque, se z=-(x+y)/2 eh SEMPRE solucao disso (independente de
>>> "inteiros" ou nao), quer dizer que essa coisa horrorosa, passando tudo para
>>> o outro lado, tem z+(x+y)/2 como fator, ou seja, x+y+2z como fator.
>>> Analogamente, vai ter 2x+y+z e x+2y+z tambem!
>>>
>>> Em suma, a gente pode voltar na primeira equacao com a sua ideia, jogar
>>> tudo para a esquerda (exceto pelo 1 chato que nao aparece na sua
>>> expressao), e fatorar. Vejamos... Acho que fica assim:
>>>
>>> 1/2*(2x+y+z)(x+2y+z)(x+y+2z)=1
>>> (2x+y+z)(x+2y+z)(x+y+2z)=2
>>>
>>> Confiram se eu nao errei contas... Mas agora ficou **bem** facil! :D
>>>
>>> Abraco, Ralph.
>>>
>>> 2018-03-19 14:33 GMT-03:00 Pedro José :
>>>
 Bom dia!

 Estou só conjecturando. Pois, não consegui nenhuma restrição.
 A única coisa que consegui, mas não me adiantou de nada, é que:
 x,y pares ou x,y ímpares e z = -(x+y)/2 é solução de

 *(x + y)(y + z)(z + x)/2 + (x + y + z)3 =  – xyz*
 Também, não consegui provar que é a única família de solução da
 equação acima para inteiros.

 Em 19 de março de 2018 14:14, Claudio Buffara <
 claudio.buff...@gmail.com> escreveu:

> Podem existir soluções não triviais envolvendo inteiros negativos.
>
> 2018-03-19 10:17 GMT-03:00 Pedro José :
>
>> Bom dia!
>>
>> Poderia postar a solução? Não consegui achar nenhuma restrição para
>> trabalhar num subconjunto  pequeno dos inteiros.
>> Creio que vá ser apenas a trivial (0,0,1) e suas permutações.
>>
>> grato,
>> PJMS
>>
>> Em 13 de março de 2018 20:19, Douglas Oliveira de Lima <
>> profdouglaso.del...@gmail.com> escreveu:
>>
>>> Essa achei legal e estou postando.
>>>
>>> *Resolva nos inteiros a seguinte equação:  (x + y)(y + z)(z + x)/2 +
>>> 

[obm-l] Re: [obm-l] Re: [obm-l] Re: [obm-l] Re: [obm-l] Re: [obm-l] Teoria dos números

2018-03-20 Por tôpico Pedro José
Boa tarde!

Ralph,
parabéns pela sua resolução.
Já, eu, caminhei por caminhos bem mais tortuosos.
Embora extremamente deselegante é uma solução.

Se xo,yo,zo é uma solução, temos que pelo menos duas incógnitas têm a mesma
paridade.
Como o problema é simétrico, sem perda de generalidade, vamos supor que xo
e yo tenham a mesma paridade.
então podemos escrever zo = -(xo+yo)/2 + h, com h inteiro.
Sabendo-se que : [xo,yo, -(xo+yo)/2] é solução da equação sem o "1".

e substituindo na equação original:

h^3 + 2(xo+yo)h^2+ (3/4 (x+y)^2 + xy) h - 1=0.

Como os coeficientes são inteiros as únicas possibilidades de h inteiro são
1 e -1.

h=1.

Seja a= xo +yo

3a^2 + 8a + 4xoyo=0

xoyo >0, temos que 8|a| > 3a^2 ==> |a| < 8/3 ==>|a|=2, portanto xo= -1 e
yo=-1(não podem ser positivos). Temos z=-(xo+yo)/2+h=0. (-1,-1,2) e suas
permutações são soluções.

xo.yo = 0 temos a=0 ou a= -8/3 (não atende) ==> xo=yo=0, z= 1. (0,0,1) e
suas permutações são soluções.

xo.y0 <0

3a^2 + 8a + 4xoyo=0

para ter solução a inteiro:Δ = (8 + 6i)^2  ==> 64 - 48 xoyo =  64 + 96 i +
36 i^2 ==> xoyo = - (2i + 3î^2/4), com i par.

a= i e xoyo = -(2i+ 3i^2/4) então xo e yo são soluções da equação t^2 -it -
(2i +3i^2/4) = 0. i =2k; t^2 -2kt-(4k+3k^2)=0

Δ = 4k^2 +16k + 12k^2 = 16k(k+1), que nunca será um quadrado perfeito com
k<>0. Então não há soluções inteiras. (k=0, recaí em xoyo=0)

h=-1

Seja a= xo+yo

-3a^2 +8a -( 4xoyo - 8) = 0

4xoyo> 8 ==> 8a >3a^2; a <=2; absurdo não atende 4xoyo>8.

4xoyo-8=0

temos que a=o, não há inteiros que xoyo=2 e xo+yo=0.

4xoyo - 8 < 0

Δ = (8 + 6i)^2 ; 64 - 48xoyo + 32 = 64 + 96 i + 36 i^2;  xoyo = -2/3 + 2i +
3/4i^2, xoyo não pertence aos inteiros não há solução.


Ficam apenas: (0,0,1) ; (0,1,0); (1,0,0) ; (-1,-1,2); (-1,2;-1); (2,-1,-1)

Ralph,

Fiz esse salseiro todo, ao invés de fatorar. E olha, que ontem estava
orgulhoso de ter achado a solução.


Saudações,
PJMS



Em 20 de março de 2018 12:10, Pedro José  escreveu:

> É acabou me ajudando. Resolvi de uma outra forma, mais complicada, usando
> a fórmula. Quando tiver um tempo eu posto.
>
>
> Em 19 de mar de 2018 21:02, "Ralph Teixeira"  escreveu:
>
>> Opa, opa, opa! Pedro, voce achou uma formula assim generica, z=-(x+y)/2,
>> que resolve esta equacao? Beleza, excelente ideia, temos um caminho!
>>
>> Porque, se z=-(x+y)/2 eh SEMPRE solucao disso (independente de "inteiros"
>> ou nao), quer dizer que essa coisa horrorosa, passando tudo para o outro
>> lado, tem z+(x+y)/2 como fator, ou seja, x+y+2z como fator. Analogamente,
>> vai ter 2x+y+z e x+2y+z tambem!
>>
>> Em suma, a gente pode voltar na primeira equacao com a sua ideia, jogar
>> tudo para a esquerda (exceto pelo 1 chato que nao aparece na sua
>> expressao), e fatorar. Vejamos... Acho que fica assim:
>>
>> 1/2*(2x+y+z)(x+2y+z)(x+y+2z)=1
>> (2x+y+z)(x+2y+z)(x+y+2z)=2
>>
>> Confiram se eu nao errei contas... Mas agora ficou **bem** facil! :D
>>
>> Abraco, Ralph.
>>
>> 2018-03-19 14:33 GMT-03:00 Pedro José :
>>
>>> Bom dia!
>>>
>>> Estou só conjecturando. Pois, não consegui nenhuma restrição.
>>> A única coisa que consegui, mas não me adiantou de nada, é que:
>>> x,y pares ou x,y ímpares e z = -(x+y)/2 é solução de
>>>
>>> *(x + y)(y + z)(z + x)/2 + (x + y + z)3 =  – xyz*
>>> Também, não consegui provar que é a única família de solução da equação
>>> acima para inteiros.
>>>
>>> Em 19 de março de 2018 14:14, Claudio Buffara >> > escreveu:
>>>
 Podem existir soluções não triviais envolvendo inteiros negativos.

 2018-03-19 10:17 GMT-03:00 Pedro José :

> Bom dia!
>
> Poderia postar a solução? Não consegui achar nenhuma restrição para
> trabalhar num subconjunto  pequeno dos inteiros.
> Creio que vá ser apenas a trivial (0,0,1) e suas permutações.
>
> grato,
> PJMS
>
> Em 13 de março de 2018 20:19, Douglas Oliveira de Lima <
> profdouglaso.del...@gmail.com> escreveu:
>
>> Essa achei legal e estou postando.
>>
>> *Resolva nos inteiros a seguinte equação:  (x + y)(y + z)(z + x)/2 +
>> (x + y + z)3 = 1 – xyz* .
>>
>> Abraço do
>> Douglas Oliveira
>>
>> --
>> Esta mensagem foi verificada pelo sistema de antivírus e
>> acredita-se estar livre de perigo.
>
>
>
> --
> Esta mensagem foi verificada pelo sistema de antivírus e
> acredita-se estar livre de perigo.
>


 --
 Esta mensagem foi verificada pelo sistema de antivírus e
 acredita-se estar livre de perigo.

>>>
>>>
>>> --
>>> Esta mensagem foi verificada pelo sistema de antivírus e
>>> acredita-se estar livre de perigo.
>>>
>>
>>
>> --
>> Esta mensagem foi verificada pelo sistema de antivírus e
>> acredita-se estar livre de perigo.
>
>

-- 
Esta mensagem foi verificada pelo sistema de antiv�rus e
 acredita-se estar livre de perigo.



[obm-l] Re: [obm-l] Re: [obm-l] Re: [obm-l] Re: [obm-l] Re: [obm-l] Teoria dos números

2018-03-20 Por tôpico Pedro José
É acabou me ajudando. Resolvi de uma outra forma, mais complicada, usando a
fórmula. Quando tiver um tempo eu posto.

Em 19 de mar de 2018 21:02, "Ralph Teixeira"  escreveu:

> Opa, opa, opa! Pedro, voce achou uma formula assim generica, z=-(x+y)/2,
> que resolve esta equacao? Beleza, excelente ideia, temos um caminho!
>
> Porque, se z=-(x+y)/2 eh SEMPRE solucao disso (independente de "inteiros"
> ou nao), quer dizer que essa coisa horrorosa, passando tudo para o outro
> lado, tem z+(x+y)/2 como fator, ou seja, x+y+2z como fator. Analogamente,
> vai ter 2x+y+z e x+2y+z tambem!
>
> Em suma, a gente pode voltar na primeira equacao com a sua ideia, jogar
> tudo para a esquerda (exceto pelo 1 chato que nao aparece na sua
> expressao), e fatorar. Vejamos... Acho que fica assim:
>
> 1/2*(2x+y+z)(x+2y+z)(x+y+2z)=1
> (2x+y+z)(x+2y+z)(x+y+2z)=2
>
> Confiram se eu nao errei contas... Mas agora ficou **bem** facil! :D
>
> Abraco, Ralph.
>
> 2018-03-19 14:33 GMT-03:00 Pedro José :
>
>> Bom dia!
>>
>> Estou só conjecturando. Pois, não consegui nenhuma restrição.
>> A única coisa que consegui, mas não me adiantou de nada, é que:
>> x,y pares ou x,y ímpares e z = -(x+y)/2 é solução de
>>
>> *(x + y)(y + z)(z + x)/2 + (x + y + z)3 =  – xyz*
>> Também, não consegui provar que é a única família de solução da equação
>> acima para inteiros.
>>
>> Em 19 de março de 2018 14:14, Claudio Buffara 
>> escreveu:
>>
>>> Podem existir soluções não triviais envolvendo inteiros negativos.
>>>
>>> 2018-03-19 10:17 GMT-03:00 Pedro José :
>>>
 Bom dia!

 Poderia postar a solução? Não consegui achar nenhuma restrição para
 trabalhar num subconjunto  pequeno dos inteiros.
 Creio que vá ser apenas a trivial (0,0,1) e suas permutações.

 grato,
 PJMS

 Em 13 de março de 2018 20:19, Douglas Oliveira de Lima <
 profdouglaso.del...@gmail.com> escreveu:

> Essa achei legal e estou postando.
>
> *Resolva nos inteiros a seguinte equação:  (x + y)(y + z)(z + x)/2 +
> (x + y + z)3 = 1 – xyz* .
>
> Abraço do
> Douglas Oliveira
>
> --
> Esta mensagem foi verificada pelo sistema de antivírus e
> acredita-se estar livre de perigo.



 --
 Esta mensagem foi verificada pelo sistema de antivírus e
 acredita-se estar livre de perigo.

>>>
>>>
>>> --
>>> Esta mensagem foi verificada pelo sistema de antivírus e
>>> acredita-se estar livre de perigo.
>>>
>>
>>
>> --
>> Esta mensagem foi verificada pelo sistema de antivírus e
>> acredita-se estar livre de perigo.
>>
>
>
> --
> Esta mensagem foi verificada pelo sistema de antivírus e
> acredita-se estar livre de perigo.

-- 
Esta mensagem foi verificada pelo sistema de antiv�rus e
 acredita-se estar livre de perigo.



[obm-l] Re: [obm-l] Re: [obm-l] Re: [obm-l] Re: [obm-l] RE: [obm-l] Re: [obm-l] RE: [obm-l] TEORIA DOS NÚMEROS

2012-03-25 Por tôpico Marcelo Salhab Brogliato
Vamos lá:
333^555 + 555^333 = 111^555 * 3^555 + 111^333 * 5^333 = 111^333 * 111^222 *
3^555 + 111^333 * 5^333

--

Como 97 é primo, pelo pequeno teorema de fermat, temos que: x^96 == 1 (mod
97).

Como 111 == 15 (mod 96) e 111 == 14 (mod 97), temos que: 111^111 == 14^15
(mod 97).
Mas, 14^2 == 2 (mod 97), então: 14^15 == 2^7 * 14 (mod 97) == 46

Então: 111^111 == 46 (mod 97).
Assim, 111^222 == 46^2 == 79 (mod 97).
Por fim, 111^333 == 46^3 == 79*46 == 45 (mod 97).

--

Voltando, temos: 111^333 * 111^222 * 3^555 + 111^333 * 5^333 == 45 * 79 *
3^555 + 45 * 5^333 (mod 97).

--

Como 555 == 75 (mod 96), temos: 3^555 == 3^75 (mod 97)
3^5 = 243 == 49 (mod 97), então: 3^75 == 49^15 (mod 97)
49^2 == 73 (mod 97), então: 49^15 == 73^7 * 49 (mod 97)
73^2 == 91 (mod 97), então: 73^7 * 49 == 91^3 * 73 * 49 == 70 (mod 97)
Enfim, 3^555 == 70 (mod 97)

Como 333 == 45 (mod 96), temos 5^333 == 5^45 (mod 97)
5^4 = 625 == 43 (mod 97), então: 5^45 == 43^11 * 5 (mod 97)
43^2 == 6 (mod 97), então: 43^11 * 5 == 6^5 * 43 * 5 = 36*36*6*43*5 == 45
(mod 97)
Enfim, 5^333 == 45 (mod 97)

--

Novamente, voltando, temos: 45 * 79 * 70 + 45 * 45 (mod 97).
Agora ficou fácil, hehe. =]

Concluindo, ficamos com 333^555 + 555^333 == 33 (mod 97).

Abraços,
Salhab




2012/3/24 Vanderlei * vanderma...@gmail.com

 *é mesmo? Então o enunciado está errado? Essa questão está no material do
 Poliedro, caderno do ITA número 1.

 *
 Em 24 de março de 2012 18:59, Bernardo Freitas Paulo da Costa 
 bernardo...@gmail.com escreveu:

 2012/3/24 Vanderlei * vanderma...@gmail.com:
  Pois é caro João, eu também cheguei nesse seu resto de 45. Mas vamos
  continuar na luta, alguma saída deve ter! A sua ideia parece ser muito
 boa,
  uma vez que o primeiro resultado é verdadeiro!
 O Maple 10 acha que

 333^555 + 555^333 mod 97 = 33...

 --
 Bernardo Freitas Paulo da Costa

 =
 Instruções para entrar na lista, sair da lista e usar a lista em
 http://www.mat.puc-rio.br/~obmlistas/obm-l.html
 =





[obm-l] Re: [obm-l] Re: [obm-l] Re: [obm-l] Re: [obm-l] Re: [obm-l] RE: [obm-l] Re: [obm-l] RE: [obm-l] TEORIA DOS NÚMEROS

2012-03-25 Por tôpico Vanderlei *
*obrigado Marcelo! Então o enunciado está errado mesmo! 97 não divide a
soma!

*
2012/3/25 Marcelo Salhab Brogliato msbro...@gmail.com

  Vamos lá:
 333^555 + 555^333 = 111^555 * 3^555 + 111^333 * 5^333 = 111^333 * 111^222
 * 3^555 + 111^333 * 5^333

 --

 Como 97 é primo, pelo pequeno teorema de fermat, temos que: x^96 == 1 (mod
 97).

 Como 111 == 15 (mod 96) e 111 == 14 (mod 97), temos que: 111^111 == 14^15
 (mod 97).
 Mas, 14^2 == 2 (mod 97), então: 14^15 == 2^7 * 14 (mod 97) == 46

 Então: 111^111 == 46 (mod 97).
 Assim, 111^222 == 46^2 == 79 (mod 97).
 Por fim, 111^333 == 46^3 == 79*46 == 45 (mod 97).

 --

 Voltando, temos: 111^333 * 111^222 * 3^555 + 111^333 * 5^333 == 45 * 79 *
 3^555 + 45 * 5^333 (mod 97).

 --

 Como 555 == 75 (mod 96), temos: 3^555 == 3^75 (mod 97)
 3^5 = 243 == 49 (mod 97), então: 3^75 == 49^15 (mod 97)
 49^2 == 73 (mod 97), então: 49^15 == 73^7 * 49 (mod 97)
 73^2 == 91 (mod 97), então: 73^7 * 49 == 91^3 * 73 * 49 == 70 (mod 97)
 Enfim, 3^555 == 70 (mod 97)

 Como 333 == 45 (mod 96), temos 5^333 == 5^45 (mod 97)
 5^4 = 625 == 43 (mod 97), então: 5^45 == 43^11 * 5 (mod 97)
 43^2 == 6 (mod 97), então: 43^11 * 5 == 6^5 * 43 * 5 = 36*36*6*43*5 == 45
 (mod 97)
 Enfim, 5^333 == 45 (mod 97)

 --

 Novamente, voltando, temos: 45 * 79 * 70 + 45 * 45 (mod 97).
 Agora ficou fácil, hehe. =]

 Concluindo, ficamos com 333^555 + 555^333 == 33 (mod 97).

 Abraços,
 Salhab




 2012/3/24 Vanderlei * vanderma...@gmail.com

 *é mesmo? Então o enunciado está errado? Essa questão está no material
 do Poliedro, caderno do ITA número 1.

 *
 Em 24 de março de 2012 18:59, Bernardo Freitas Paulo da Costa 
 bernardo...@gmail.com escreveu:

 2012/3/24 Vanderlei * vanderma...@gmail.com:
  Pois é caro João, eu também cheguei nesse seu resto de 45. Mas vamos
  continuar na luta, alguma saída deve ter! A sua ideia parece ser muito
 boa,
  uma vez que o primeiro resultado é verdadeiro!
 O Maple 10 acha que

 333^555 + 555^333 mod 97 = 33...

 --
 Bernardo Freitas Paulo da Costa

 =
 Instruções para entrar na lista, sair da lista e usar a lista em
 http://www.mat.puc-rio.br/~obmlistas/obm-l.html
 =






[obm-l] Re: [obm-l] Re: [obm-l] RE: [obm-l] Re: [obm-l] RE: [obm-l] TEORIA DOS NÚMEROS

2012-03-24 Por tôpico Bernardo Freitas Paulo da Costa
2012/3/24 Vanderlei * vanderma...@gmail.com:
 Pois é caro João, eu também cheguei nesse seu resto de 45. Mas vamos
 continuar na luta, alguma saída deve ter! A sua ideia parece ser muito boa,
 uma vez que o primeiro resultado é verdadeiro!
O Maple 10 acha que

333^555 + 555^333 mod 97 = 33...

-- 
Bernardo Freitas Paulo da Costa

=
Instruções para entrar na lista, sair da lista e usar a lista em
http://www.mat.puc-rio.br/~obmlistas/obm-l.html
=


[obm-l] Re: [obm-l] Re: [obm-l] Re: [obm-l] RE: [obm-l] Re: [obm-l] RE: [obm-l] TEORIA DOS NÚMEROS

2012-03-24 Por tôpico Vanderlei *
*é mesmo? Então o enunciado está errado? Essa questão está no material do
Poliedro, caderno do ITA número 1.

*
Em 24 de março de 2012 18:59, Bernardo Freitas Paulo da Costa 
bernardo...@gmail.com escreveu:

 2012/3/24 Vanderlei * vanderma...@gmail.com:
  Pois é caro João, eu também cheguei nesse seu resto de 45. Mas vamos
  continuar na luta, alguma saída deve ter! A sua ideia parece ser muito
 boa,
  uma vez que o primeiro resultado é verdadeiro!
 O Maple 10 acha que

 333^555 + 555^333 mod 97 = 33...

 --
 Bernardo Freitas Paulo da Costa

 =
 Instruções para entrar na lista, sair da lista e usar a lista em
 http://www.mat.puc-rio.br/~obmlistas/obm-l.html
 =



[obm-l] Re: [obm-l] Re: [obm-l] Re: [obm-l] Re: [obm-l] Re: [obm-l] Teoria dos números

2010-12-21 Por tôpico Willy George do Amaral Petrenko
Claro, claro, foi um erro de tipografia.

2010/12/21 Henrique Rennó henrique.re...@gmail.com

 Minha dúvida é sobre o expoente do termo a^'pq - 2q', não seria a^'pq - 2p'
 ?

 Em 18/12/10, Willy George do Amaral Petrenkowgapetre...@gmail.com
 escreveu:
  Escreva num papel e veja algum caso particular. Por exemplo:
 
  a^5 + a^4 + a^3 + a^2 + a + 1 = a^3*(a^2 + a + 1) + a^2 + a + 1 = (a^3 +
  1)*(a^2 + a + 1)
 
  Repare que se n = 9, a primeira parcela ficaria (a^6 + a^3 + 1).
 
  2010/12/17 Henrique Rennó henrique.re...@gmail.com
 
  Não entendi como a^'n-1' + a^'n-2' + ... + a + 1 = (a^'pq - p' + a^'pq
  - 2q' + ... + a^p + 1)*(a^'p - 1' + a^'p - 2' + ... + a + 1).
 
  Em 17/12/10, Willy George do Amaral Petrenkowgapetre...@gmail.com
  escreveu:
   Observe que a^n - 1 = (a - 1)*(a^'n-1' + a^'n-2' + ... + a + 1). Se
 a^n
   -
  1
   é primo então a 1a parcela deve ser 1 (a 2a não pode para a0), e
 então
   a
  =
   2.
  
   Agora observe que se n = p*q então a^'n-1' + a^'n-2' + ... + a + 1 =
  (a^'pq
   - p' + a^'pq - 2q' + ... + a^p + 1)*(a^'p - 1' + a^'p - 2' + ... + a +
  1),
   ambas parcelas maiores que 1 para p,q 1.
   n composto =  a^n - 1 composto logo a^n - 1 primo = n primo.
  
  
  
   2010/12/16 marcone augusto araújo borges marconeborge...@hotmail.com
 
  
Mostre que se a e n são inteiros positivos,com n = 2 ,tais que a^n
 -
   1
  é
   primo,então necessariamente a = 2 e n é primo.
  
  
 
 
  --
  Henrique
 
 
 =
  Instruções para entrar na lista, sair da lista e usar a lista em
  http://www.mat.puc-rio.br/~obmlistas/obm-l.html
 
 =
 
 


 --
 Henrique

 =
 Instruções para entrar na lista, sair da lista e usar a lista em
 http://www.mat.puc-rio.br/~obmlistas/obm-l.html
 =



[obm-l] Re: [obm-l] RE: [obm-l] Re: [obm-l] Re : [obm-l] Re: [obm-l] RE: [obm-l] Teoria dos números (2 questões simples)

2009-08-24 Por tôpico luiz silva


Ola,
 
Tente isso..acho q funciona .
 
1) b^2+ab+1 = 0 mod (a^2+ab+1)
 
2) a^2+ab+1= 0 mod (a^2+ab+1)
 
Substiutua (2) em (1)
 
Abs
Felipe
 
--- Em sex, 21/8/09, marcone augusto araújo borges 
marconeborge...@hotmail.com escreveu:


De: marcone augusto araújo borges marconeborge...@hotmail.com
Assunto: [obm-l] RE: [obm-l] Re: [obm-l] Re: [obm-l] Re: [obm-l] RE: [obm-l] 
Teoria dos números (2 questões simples)
Para: obm-l@mat.puc-rio.br
Data: Sexta-feira, 21 de Agosto de 2009, 21:38




#yiv1877891977 #yiv1193512529 .hmmessage P
{
margin:0px;padding:0px;}
#yiv1877891977 #yiv1193512529 {
font-size:10pt;font-family:Verdana;}

Hugo esclareceu,obrigado.Mas o Diogo soicitou ajuda em outra questão: se 
a^2+ab+1 divide b^2+ab+1 então a=b.Alguém poderia ajudar?
 


Date: Fri, 21 Aug 2009 16:34:51 -0700
From: luizfelipec...@yahoo.com.br
Subject: [obm-l] Re: [obm-l] Re: [obm-l] Re: [obm-l] RE: [obm-l] Teoria dos 
números (2 questões simples)
To: obm-l@mat.puc-rio.br






Hugo,
 
Valeu!!
Abs
Felipe

--- Em sex, 21/8/09, Hugo Fernando Marques Fernandes hfernande...@gmail.com 
escreveu:


De: Hugo Fernando Marques Fernandes hfernande...@gmail.com
Assunto: [obm-l] Re: [obm-l] Re: [obm-l] RE: [obm-l] Teoria dos números (2 
questões simples)
Para: obm-l@mat.puc-rio.br
Data: Sexta-feira, 21 de Agosto de 2009, 16:14



Todos os números são da forma 3k+1, 3k ou 3k-1. Como 3k não é primo, k1, então 
todos os primos maiores que 3 são da forma 3k+1 ou 3k-1.
 
Abraços.
 
Hugo.


2009/8/21 luiz silva luizfelipec...@yahoo.com.br






Ola Marcone,
 
Pq vc trabalha com primos da forma 3k+1 ou 3k-1..creio que nem todos os primos 
podem ser representados nesta forma...O correto não seria 2k+1 ou 2k-1 ??Ou 
então, se quiser representar primos maiores que 3, não seria correto trabalhar 
com primos da forma 3+2k?
 
Abs
Felipe

--- Em qui, 20/8/09, marcone augusto araújo borges 
marconeborge...@hotmail.com escreveu:


De: marcone augusto araújo borges marconeborge...@hotmail.com
Assunto: [obm-l] RE: [obm-l] Teoria dos números (2 questões simples)
Para: obm-l@mat.puc-rio.br
Data: Quinta-feira, 20 de Agosto de 2009, 21:10 



Olá,Diogo.Um comentário singelo:o único primo múltiplo de 3 é o próprio 3.Se a 
é primo e diferente de 3, então a=3k+1 ou a=3k-1,dai a^2=3p+1 e a^2 +2=3q, o 
que é uma contradição(pois a^2+2 é primo).Portanto a=3.Se eu estiver 
errado,certamente alguem irá corrigir.Um abraço.
 


Date: Thu, 20 Aug 2009 15:15:39 -0700
From: diog...@yahoo.com.br
Subject: [obm-l] Teoria dos números (2 questões simples)
To: obm-l@mat.puc-rio.br




Teoria dos números (2 questões simples)?
1. Mostre que se (a) e (a² + 2) são ambos primos então a=3
2. Mostre que se (a² +ab +1) divide (b² +ab + 1) então a=b.


Se puder ajudar, agradeço.


Veja quais são os assuntos do momento no Yahoo! + Buscados: Top 10 - 
Celebridades - Música - Esportes


Novo Internet Explorer 8: mais rápido e muito mais seguro. Baixe agora, é 
grátis! 





Veja quais são os assuntos do momento no Yahoo! + Buscados: Top 10 - 
Celebridades - Música - Esportes



Veja quais são os assuntos do momento no Yahoo! + Buscados: Top 10 - 
Celebridades - Música - Esportes


Instale o novo Internet Explorer 8 versão especial para o MSN. Download aqui 


  

Veja quais são os assuntos do momento no Yahoo! +Buscados
http://br.maisbuscados.yahoo.com

[obm-l] RE: [obm-l] Re: [obm -l] Re: [obm-l] Re: [obm-l] RE: [obm-l] Teoria dos números ( 2 questões simples)

2009-08-21 Por tôpico marcone augusto araújo borges

Hugo esclareceu,obrigado.Mas o Diogo soicitou ajuda em outra questão: se 
a^2+ab+1 divide b^2+ab+1 então a=b.Alguém poderia ajudar?
 


Date: Fri, 21 Aug 2009 16:34:51 -0700
From: luizfelipec...@yahoo.com.br
Subject: [obm-l] Re: [obm-l] Re: [obm-l] Re: [obm-l] RE: [obm-l] Teoria dos 
números (2 questões simples)
To: obm-l@mat.puc-rio.br






Hugo,
 
Valeu!!
Abs
Felipe

--- Em sex, 21/8/09, Hugo Fernando Marques Fernandes hfernande...@gmail.com 
escreveu:


De: Hugo Fernando Marques Fernandes hfernande...@gmail.com
Assunto: [obm-l] Re: [obm-l] Re: [obm-l] RE: [obm-l] Teoria dos números (2 
questões simples)
Para: obm-l@mat.puc-rio.br
Data: Sexta-feira, 21 de Agosto de 2009, 16:14



Todos os números são da forma 3k+1, 3k ou 3k-1. Como 3k não é primo, k1, então 
todos os primos maiores que 3 são da forma 3k+1 ou 3k-1.
 
Abraços.
 
Hugo.


2009/8/21 luiz silva luizfelipec...@yahoo.com.br






Ola Marcone,
 
Pq vc trabalha com primos da forma 3k+1 ou 3k-1..creio que nem todos os primos 
podem ser representados nesta forma...O correto não seria 2k+1 ou 2k-1 ??Ou 
então, se quiser representar primos maiores que 3, não seria correto trabalhar 
com primos da forma 3+2k?
 
Abs
Felipe

--- Em qui, 20/8/09, marcone augusto araújo borges 
marconeborge...@hotmail.com escreveu:


De: marcone augusto araújo borges marconeborge...@hotmail.com
Assunto: [obm-l] RE: [obm-l] Teoria dos números (2 questões simples)
Para: obm-l@mat.puc-rio.br
Data: Quinta-feira, 20 de Agosto de 2009, 21:10 



Olá,Diogo.Um comentário singelo:o único primo múltiplo de 3 é o próprio 3.Se a 
é primo e diferente de 3, então a=3k+1 ou a=3k-1,dai a^2=3p+1 e a^2 +2=3q, o 
que é uma contradição(pois a^2+2 é primo).Portanto a=3.Se eu estiver 
errado,certamente alguem irá corrigir.Um abraço.
 


Date: Thu, 20 Aug 2009 15:15:39 -0700
From: diog...@yahoo.com.br
Subject: [obm-l] Teoria dos números (2 questões simples)
To: obm-l@mat.puc-rio.br




Teoria dos números (2 questões simples)?
1. Mostre que se (a) e (a² + 2) são ambos primos então a=3
2. Mostre que se (a² +ab +1) divide (b² +ab + 1) então a=b.


Se puder ajudar, agradeço.


Veja quais são os assuntos do momento no Yahoo! + Buscados: Top 10 - 
Celebridades - Música - Esportes


Novo Internet Explorer 8: mais rápido e muito mais seguro. Baixe agora, é 
grátis! 





Veja quais são os assuntos do momento no Yahoo! + Buscados: Top 10 - 
Celebridades - Música - Esportes



Veja quais são os assuntos do momento no Yahoo! + Buscados: Top 10 - 
Celebridades - Música - Esportes
_
Novo Internet Explorer 8. Baixe agora, é grátis!
http://brasil.microsoft.com.br/IE8/mergulhe/?utm_source=MSN%3BHotmailutm_medium=Taglineutm_campaign=IE8

[obm-l] Re: [obm-l] Re: [obm-l] Re: [obm-l] Re: [obm-l] Re: [obm-l] teoria dos números

2009-03-31 Por tôpico Bernardo Freitas Paulo da Costa
2009/3/30 Ralph Teixeira ralp...@gmail.com:

 Tá, eu confesso: comprei o Scientific Workplace, que faz estas contas
 na boa. Tenho certeza que há outros pacotes matemáticos grátis por aí
 que também fazem estas contas grandes.

 Abraço,
        Ralph

Eu usei o bc (gratis, vem com provavelmente todos os linux do mundo,
em linha de comando, basta escrever 5^50 e ele mostra quanto vale, dai
copia de uma linha pra outra botando +) pra fazer a conta, mas
confesso que eu primeiro resolvi o problema de somar tudo varias vezes
antes de pedir pra ele confirmar que o resto mod 9 era o que eu tinha
encontrado. O que sempre é bom de fazer pra ter certeza de nao ter
deixado passar uma besteirinha !

-- 
Bernardo Freitas Paulo da Costa

=
Instruções para entrar na lista, sair da lista e usar a lista em
http://www.mat.puc-rio.br/~obmlistas/obm-l.html
=